Mathcenter Forum  

Go Back   Mathcenter Forum > ค้นหาในห้อง
สมัครสมาชิก คู่มือการใช้ รายชื่อสมาชิก ปฏิทิน ค้นหา ข้อความวันนี้ ทำเครื่องหมายอ่านทุกห้องแล้ว

แสดงผลลัพธ์ตั้งแต่ 1 ถึง 25 จากทั้งหมด 56
ใช้เวลาค้นหา 0.01 วินาที.
ค้นหา: ข้อความของคุณ: Tohn
ห้อง: คณิตศาสตร์อุดมศึกษา 24 มิถุนายน 2013, 02:05
คำตอบ: 0
เปิดอ่าน: 938
ข้อความของคุณ Tohn
ช่วยทีคับ Finite Groups Perfect Order Subsets

Let G be a finite group and all the Sylow subgroups of G are cyclic, then
$G=<x, y|x^m =1= y^n , xyx^{−1}= y^r>$ where $0 ≤ r<m, r^n \equiv 1 \pmod {m}$, $m$ is odd, $gcd(m, n(r − 1)) = 1$,...
ห้อง: คณิตศาสตร์อุดมศึกษา 06 กันยายน 2012, 21:26
คำตอบ: 6
เปิดอ่าน: 1,810
ข้อความของคุณ Tohn
มีเท่านี้อ่าครับผม

มีเท่านี้อ่าครับผม
ห้อง: คณิตศาสตร์อุดมศึกษา 06 กันยายน 2012, 17:50
คำตอบ: 6
เปิดอ่าน: 1,810
ข้อความของคุณ Tohn
$W^o = \{f \in L(V,F) : f(v)=0$ $\forall ...

$W^o = \{f \in L(V,F) : f(v)=0$ $\forall v \in W \}.$

3.Let $V$ be a finite dimensional over a field $F$ and $W_1 ,W_2$ are subspaces of $V$. Prove that if ${W_1}^o = {W_2}^0$ ,then...
ห้อง: คณิตศาสตร์อุดมศึกษา 05 กันยายน 2012, 22:04
คำตอบ: 6
เปิดอ่าน: 1,810
ข้อความของคุณ Tohn
Linear transformation คับ

1.) Let $f,g \in L(V,F).$ Prove that for all $v \in V , f(v)=0 -> g(v)=0$ ,then $g=af$ for some $a \in F.$

2.) Let $V$ and $W$ be vector space over field $F$ and $W \neq \{0\}.$ If $v \in V$ is such...
ห้อง: ทฤษฎีจำนวน 24 พฤศจิกายน 2011, 08:14
คำตอบ: 4
เปิดอ่าน: 2,362
ข้อความของคุณ Tohn
ขอบคุณมากๆคับ ทุกคนเลย

ขอบคุณมากๆคับ ทุกคนเลย
ห้อง: ทฤษฎีจำนวน 15 พฤศจิกายน 2011, 05:28
คำตอบ: 4
เปิดอ่าน: 2,362
ข้อความของคุณ Tohn
ปัญหา x^p+y^p

ให้ $p$ เป็นจำนวนเฉพาะ และ $m\geqslant 2$ เป็นจำนวนเต็ม ถ้า $ \frac{x^p+y^p}{2}={\left(\frac{x+y}{2}\right)}^m $ มีผลเฉลย $(x,y) \neq (1,1)$ เป็นจำนวนเต็มบวกแล้ว $m = p$
ห้อง: ทฤษฎีจำนวน 13 พฤศจิกายน 2011, 21:02
คำตอบ: 2
เปิดอ่าน: 2,008
ข้อความของคุณ Tohn
ปัญหาเกี่ยวกับ n \mid (2^n+1)

จงพิสูจน์ว่า จะมีจำนวนเต็มบวก $n={p_{1}}^2p_{2}p_{3}\ldots p_{2000}$ โดยที่ $p_{1} , p_{2} , \ldots , p_{2000}$ เป็นจำนวนเฉพาะที่แตกต่างกัน ซึ่ง $n \mid (2^n+1) $
ห้อง: ทฤษฎีจำนวน 31 ตุลาคม 2011, 22:34
คำตอบ: 4
เปิดอ่าน: 2,789
ข้อความของคุณ Tohn
บางทีผมอาจแปลโจทย์ไม่ดี นี่คับต้นฉบับ Prove that...

บางทีผมอาจแปลโจทย์ไม่ดี นี่คับต้นฉบับ Prove that the equation $x^n + y^n = (x + y)^m$ has a unique solution
in integers satisfying $x > y,m > 1, n > 1$
ห้อง: ทฤษฎีจำนวน 31 ตุลาคม 2011, 07:15
คำตอบ: 4
เปิดอ่าน: 2,789
ข้อความของคุณ Tohn
พิสูจน์ unique solution ของ x^n+y^n=(x+y)^m

จงพิสูจน์ว่า $x^n+y^n=(x+y)^m$ มีผลเฉลยเป็นจำนวนเต็มบวกเพียงชุดเดียวที่สอดคล้องกับ $x>y,m>1,n>1$
ช่วยทีครับ:please:
ห้อง: ทฤษฎีจำนวน 21 กันยายน 2011, 03:43
คำตอบ: 5
เปิดอ่าน: 2,964
ข้อความของคุณ Tohn
ขอบคุณมากๆเลยครับ สำหรับวิธีคิดข้อ2...

ขอบคุณมากๆเลยครับ สำหรับวิธีคิดข้อ2 ตอนนี่ก็ยังคิดข้อ1 ไม่ออกเลย TT ไม่รู้จะเริ่มยังไงดี
ห้อง: ทฤษฎีจำนวน 21 สิงหาคม 2011, 08:53
คำตอบ: 5
เปิดอ่าน: 2,964
ข้อความของคุณ Tohn
พิสูจน์ x^n +y^n = (x+y)^m help me please

1. จงพิสูจน์ว่า $x^n+y^n=(x+y)^m$ มีผลเฉลยเป็นจำนวนเต็มเพียงชุดเดียวที่สอดคล้องกับ $x>y,m>1,n>1$
2. ถ้า $n$ เป็นจำนวนเต็มบวก ที่ $n \mid 2^n+1$ แล้ว $n=3$ หรือ $9 \mid n$
:please:
ห้อง: ทฤษฎีจำนวน 03 พฤษภาคม 2011, 04:23
คำตอบ: 6
เปิดอ่าน: 3,168
ข้อความของคุณ Tohn
ข้อแรก ลองดูอย่างนี้ครับ เพราะ$k$เป็จจำนวนคี่...

ข้อแรก ลองดูอย่างนี้ครับ เพราะ$k$เป็จจำนวนคี่
$2(1^k+2^k+...+n^k) = (n^k +1^k)+((n-1)^k+2^k) + ..... = (n+1)(....)$
$ = 2n^k+((n-1)^k+1^k)+((n-2)^k+2^k)+..=n(...)$
ห้อง: ทฤษฎีจำนวน 05 เมษายน 2011, 00:34
คำตอบ: 5
เปิดอ่าน: 2,667
ข้อความของคุณ Tohn
ไม่รู้ทำแบบนี้ได้มั้ยนะครับผม สำหรับ $n>2$ ให้...

ไม่รู้ทำแบบนี้ได้มั้ยนะครับผม
สำหรับ $n>2$
ให้ $S$ = { $k|k<n และ (k,n)=1$ } เห็นชัดว่า $|S| \not= 0$ จะได้ว่า $|S|= \phi(n)$
ให้ $d_{1} ,d_{2}, \cdots , d_{\frac{\phi(n)}{2}} \in S$ เนื่องจาก...
ห้อง: ทฤษฎีจำนวน 10 มีนาคม 2011, 13:38
คำตอบ: 1
เปิดอ่าน: 1,679
ข้อความของคุณ Tohn
รบกวนด้วยครับ

Show that if $n$ is a square-free positive integer then there do not exist relatively prime positive integers $x$ and $y$ such that $(x+y)^3$ divides $x^n+y^n.$
ห้อง: ข้อสอบในโรงเรียน ประถมปลาย 26 มกราคม 2011, 04:15
คำตอบ: 10
เปิดอ่าน: 20,148
ข้อความของคุณ Tohn
ขอทำข้อ12 อีกแบบนะครับ จากที่ทราบว่า A...

ขอทำข้อ12 อีกแบบนะครับ
จากที่ทราบว่า A รู้ว่าตัวเองเป็นจำนวนคี่มาแล้ว
และ B มั่นใจว่าตัวเองรู้ว่า จำนวนทั้งสามมีค่าต่างกันหมด B น่าจะได้เลขคู่มาครับซึ่งก็ เป็นไป ได้
2,4,6,8,10,12
ถ้า เป็น
" 2 ",...
ห้อง: คณิตศาสตร์อุดมศึกษา 22 พฤศจิกายน 2010, 23:49
คำตอบ: 3
เปิดอ่าน: 2,473
ข้อความของคุณ Tohn
รบกวนด้วยนะครับ metric space กับ จุดกับมิต

ให้ $(X ,d)$ เป็น metric space และ $x\in X $จงพิสูจน์ว่า$ x$ เป็นจุดลิมิตของ $S$ โดย $S\subseteq X$ ก็ต่อเมื่อ สำหรับทุก$ r > 0$ , $ B_d(x,r)\cap S$ เป็นเซตอนันต์

อยากให้ทำให้ดูเลยครับ :please: ...
ห้อง: ปัญหาคณิตศาสตร์ ม.ปลาย 08 พฤษภาคม 2010, 21:26
คำตอบ: 327
เปิดอ่าน: 75,956
ข้อความของคุณ Tohn
$$\int_{0}^{1} \ln{(\sqrt{1+x}+\sqrt{1-x})} \, dx...

$$\int_{0}^{1} \ln{(\sqrt{1+x}+\sqrt{1-x})} \, dx = \frac{1}{2}(\int_{0}^{1} \ln{x} \,dx + \int_{0}^{1} \ln{(1+ \sqrt{1-x^{2}})} \, dx ) $$
$$\int_{0}^{1} \ln{(1+\sqrt{1-x^{2}})} \, dx =...
ห้อง: ปัญหาคณิตศาสตร์ ม.ปลาย 08 พฤษภาคม 2010, 02:23
คำตอบ: 327
เปิดอ่าน: 75,956
ข้อความของคุณ Tohn
$$ \int_{0}^{\frac{\pi}{2}} 1\,dx...

$$ \int_{0}^{\frac{\pi}{2}} 1\,dx =\int_{0}^{\frac{\pi}{2}}( 1+2\sum_{k=1}^{1276}\cos(2kx) ) \,dx =\int_{0}^{\frac{\pi}{2}}\sum_{k=-1276}^{1276}e^{2ikx}\,dx =\int_{0}^{\frac{\pi}{2}}...
ห้อง: ปัญหาคณิตศาสตร์ทั่วไป 21 มีนาคม 2010, 23:48
คำตอบ: 2
เปิดอ่าน: 1,461
ข้อความของคุณ Tohn
ผมคิดว่าน่าจะเป็ฯงี้ จาก $r\binom{n}{r} =...

ผมคิดว่าน่าจะเป็ฯงี้
จาก $r\binom{n}{r} = n\binom{n-1}{r-1}$ จะได้
$ 0\binom{n}{0} p^n + 1\binom{n}{1} p^{n-1}q +...+ r\binom{n}{r} p^rq^{n-r} +...+ n\binom{n}{n} q^n = n[ \binom{n-1}{0} p^{n-1}q +...
ห้อง: Calculus and Analysis 16 มีนาคม 2010, 20:54
คำตอบ: 4
เปิดอ่าน: 2,457
ข้อความของคุณ Tohn
ให้ $e>0$ สมมติว่า $ a<b+e $ และ $ a>b $ เลือก...

ให้ $e>0$ สมมติว่า $ a<b+e $ และ $ a>b $ เลือก $e=\frac{1}{2}(a-b)$
โดยสมมติฐาน จะได้ $ a<b+\frac{1}{2}(a-b) = \frac{a+b}{2}$ เกิดข้อขัดแย้ง
ดังนั้น $ a\leqslant b $ คับ
ห้อง: Calculus and Analysis 16 มีนาคม 2010, 19:34
คำตอบ: 4
เปิดอ่าน: 2,457
ข้อความของคุณ Tohn
$สมมุติให้ a > b และก็ให้ e = \frac{1}{2}(a-b)...

$สมมุติให้ a > b และก็ให้ e = \frac{1}{2}(a-b) คับ จะได้ข้อขัดแย้ง$
ห้อง: ปัญหาคณิตศาสตร์ ม. ต้น 16 พฤศจิกายน 2009, 18:11
คำตอบ: 5
เปิดอ่าน: 2,078
ข้อความของคุณ Tohn
อืม ยากหง่ะ ผมทำแบบนี้ได้อ๊ะป่าว $x^3+9xy^2=26$...

อืม ยากหง่ะ ผมทำแบบนี้ได้อ๊ะป่าว
$x^3+9xy^2=26$ ....(i) และก็ $y(x^2+y^2)=5 $... (ii)
ให้ $ x=ky $ จะได้ $ y^3(k^3+9k)=26$ ..... (iii) และ $y^3(k^2+1)=5$ ... (iv) เอามาหารกัน
แก้สมการ จะได้...
ห้อง: ปัญหาคณิตศาสตร์ทั่วไป 15 กันยายน 2009, 16:37
คำตอบ: 1
เปิดอ่าน: 1,554
ข้อความของคุณ Tohn
ก็จาก...

ก็จาก อสมการค่าเฉลี่ยเลขคณิตและเรขาคณิตถ่วงน้ำหนัก
$x_1^{w_1}x_2^{w_2} \leq w_1x_1+w_2x_2$ (หาวิธีพิสูจน์ได้ไม่ยาก)
แล้วก็แทน $x_1 = e^u , x_2=e^v , w_1=\frac{1}{p} ,...
ห้อง: ปัญหาคณิตศาสตร์ทั่วไป 22 กุมภาพันธ์ 2009, 12:05
คำตอบ: 5
เปิดอ่าน: 2,392
ข้อความของคุณ Tohn
$\displaystyle{\sum_{n = 1}^{543}...

$\displaystyle{\sum_{n = 1}^{543} \frac{(2552-k)!}{(543-k)!}\frac{(543)!}{(2552)!}=\frac{(543)!}{(2552)!}\sum_{n = 1}^{543} \frac{(2552-k)!}{(543-k)!}=\frac{(543)!}{(2552)!}\sum_{n = 1}^{543}...
ห้อง: พีชคณิต 22 กุมภาพันธ์ 2009, 11:58
คำตอบ: 4
เปิดอ่าน: 2,152
ข้อความของคุณ Tohn
ผิดจริงง่ะ แล้วทำไงให้มันถูกอะครับ :please:

ผิดจริงง่ะ แล้วทำไงให้มันถูกอะครับ :please:
แสดงผลลัพธ์ตั้งแต่ 1 ถึง 25 จากทั้งหมด 56

 
ทางลัดสู่ห้อง

เวลาที่แสดงทั้งหมด เป็นเวลาที่ประเทศไทย (GMT +7) ขณะนี้เป็นเวลา 19:29


Powered by vBulletin® Copyright ©2000 - 2024, Jelsoft Enterprises Ltd.
Modified by Jetsada Karnpracha